Q3

 
kmewmewblue
Thanks Received: 1
Forum Guests
 
Posts: 57
Joined: April 18th, 2011
 
 
trophy
Most Thankful
 

Q3

by kmewmewblue Mon Nov 28, 2011 9:22 am

Why (A)? I feel this is not strong enough....
Why not (E) if (A) could be the answer.
 
extraordinary.kye
Thanks Received: 5
Forum Guests
 
Posts: 12
Joined: November 17th, 2011
 
 
 

Re: Q3

by extraordinary.kye Mon Nov 28, 2011 10:23 am

First of all, L27-30; discuss the traditional view of bipolar system. Which can helps us to eliminate answer choice (B), (C), (D) because they all bring in multipolar system. The author isn't trying to compare multipolar and bipolar system, he is just simply discussing bipolar characteristics.

We can then eliminate answer choice (D) because "nature of international relations" is too broad to be within the scope of this passage.

For correct answer : (A) ... "unstable characteristics"
we have reference in L26 "salient feature of bipolar system" which is the "characteristics" and also L28 "frequent confrontations, delibilitating armed conflict, and eventually to the capitualtion of one or the other side" resembles "unstable" in answer choice.

Hope this helps!
User avatar
 
ohthatpatrick
Thanks Received: 3808
Atticus Finch
Atticus Finch
 
Posts: 4661
Joined: April 01st, 2011
 
This post thanked 2 times.
 
 

Re: Q3

by ohthatpatrick Tue Nov 29, 2011 3:31 pm

I agree with most of how the previous poster responded, but I want to caution everyone about this question type in general.

When the stem asks,
"the author mentioned X primarily to"
"the author's reference to X serves to"
"the author uses the phrase ____ in lines # in order to"

... the question is asking about the broader function of certain detail or example. More than half the time, the correct answer to these questions is a paraphrase of the previous sentence.

For example, if a passage said:
Many types of mistruths should not be considered lies. Telling your mother that she looks beautiful, even if she doesn't, is a loving gesture, not a lie.

And there was a question asking
"the author mentions 'telling your mother that she looks beautiful' in line 37 in order to"

The correct answer would be something like
A) to illustrate the contention that not all mistruths can be treated equivalently

i.e., a paraphrase of the previous sentence

So, for Q3 here, don't get boxed into thinking that lines 27-30 are the only place we're allowed to look to support this answer. Asking "why" an author said something allows you to consider the broader point being made in that paragraph.

The 2nd paragraph is attempting to draw a contrast between bipolar and multipolar. We know this because the topic sentence of the 2nd paragraph begins "bipolar systems, on the other hand".

So the function of the 2nd paragraph is to show how bipolar systems are thought to be different from multipolar.

The big wrap-up on multipolar systems is 12-14 ... "multipolar usually results in stability"

Hence, the big contrast to draw in the 2nd paragraph is that bipolar systems are thought to be more unstable.

(E) actually contradicts this important distinction made between multipolar and bipolar.

I hope this helps.
 
shaynfernandez
Thanks Received: 5
Elle Woods
Elle Woods
 
Posts: 91
Joined: July 14th, 2011
 
 
 

Re: Q3

by shaynfernandez Thu May 17, 2012 3:40 pm

I never had a problem with E. as viable option, instead I had problems with option C. The opening of paragraph 2 shows a contrast between multipolar and bipolar systems. Option C points to an exact contrast, I can see why A would be correct but what is wrong with C?
 
hippo3717
Thanks Received: 1
Forum Guests
 
Posts: 25
Joined: October 12th, 2012
 
 
 

Re: Q3

by hippo3717 Sun Oct 28, 2012 3:40 pm

The second paragraph is all about bipolar whereas the first paragraph is all about multipolar.

Well essentially, the author is comparing and contrasting those two different systems "OVERALL," but does not specifically pick out an aspect from each approach and compare it to the other counterparts.

B,C,E all do that and they are all trap answers.
 
SilvaS630
Thanks Received: 0
Vinny Gambini
Vinny Gambini
 
Posts: 2
Joined: May 05th, 2020
 
 
 

Re: Q3

by SilvaS630 Tue Dec 22, 2020 8:24 pm

In response to someone's earlier argument, answer choice C is incorrect because it states that multipolar have more rational behavior. This is unsupported in the passage. The contrast they are drawing has to do with stability, not rationality, so that is an unsupported inference. Hope this helps the next person!
 
AbhistD667
Thanks Received: 0
Vinny Gambini
Vinny Gambini
 
Posts: 15
Joined: October 11th, 2020
 
 
 

Re: Q3

by AbhistD667 Tue Sep 21, 2021 4:53 am

ohthatpatrick Wrote:I agree with most of how the previous poster responded, but I want to caution everyone about this question type in general.

When the stem asks,
"the author mentioned X primarily to"
"the author's reference to X serves to"
"the author uses the phrase ____ in lines # in order to"

... the question is asking about the broader function of certain detail or example. More than half the time, the correct answer to these questions is a paraphrase of the previous sentence.

For example, if a passage said:
Many types of mistruths should not be considered lies. Telling your mother that she looks beautiful, even if she doesn't, is a loving gesture, not a lie.

And there was a question asking
"the author mentions 'telling your mother that she looks beautiful' in line 37 in order to"

The correct answer would be something like
A) to illustrate the contention that not all mistruths can be treated equivalently

i.e., a paraphrase of the previous sentence

So, for Q3 here, don't get boxed into thinking that lines 27-30 are the only place we're allowed to look to support this answer. Asking "why" an author said something allows you to consider the broader point being made in that paragraph.

The 2nd paragraph is attempting to draw a contrast between bipolar and multipolar. We know this because the topic sentence of the 2nd paragraph begins "bipolar systems, on the other hand".

So the function of the 2nd paragraph is to show how bipolar systems are thought to be different from multipolar.

The big wrap-up on multipolar systems is 12-14 ... "multipolar usually results in stability"

Hence, the big contrast to draw in the 2nd paragraph is that bipolar systems are thought to be more unstable.

(E) actually contradicts this important distinction made between multipolar and bipolar.

I hope this helps.


Can you please explain why C is wrong?